No bash please

Algebra Level 5

P ( x ) = x 4 + 7 x 3 + 2 x 9 \large P(x)=x^4+7x^3+2x-9

If a 1 , a 2 , a 3 , a 4 a_1,a_2,a_3,a_4 are roots of P ( x ) = 0 P(x)=0 , find the sum of coefficients of a least degree monic polynomial whose roots are a 1 a 2 a 3 , a 1 a 2 a 4 , a 1 a 3 a 4 , a 2 a 3 a 4 a_1a_2a_3,a_1a_2a_4,a_1a_3a_4,a_2a_3a_4 .


The answer is -159.

This section requires Javascript.
You are seeing this because something didn't load right. We suggest you, (a) try refreshing the page, (b) enabling javascript if it is disabled on your browser and, finally, (c) loading the non-javascript version of this page . We're sorry about the hassle.

2 solutions

Nihar Mahajan
Jun 16, 2015

Let the roots be written in form i = 1 4 a i a i = 9 a i \dfrac{\displaystyle\prod_{i=1}^4 a_i}{a_i} = \dfrac{-9}{a_i}

We introduce t i t_i which is root of our required equation such that:

t i = 9 a i a i = 9 t i i 1 i 4 t_i=\dfrac{-9}{a_i} \quad \Rightarrow a_i=\dfrac{-9}{t_i} \quad \forall \ i \ 1\leq i \leq 4

Then we have:

i 1 i 4 { ( P ( 9 t i ) = 0 ( 9 t i ) 4 + 7 ( 9 t i ) 3 + 2 ( 9 t i ) 9 = 0 9 4 t i 4 + 9 3 × 7 t i 3 + 9 × 2 t i 9 = 0 t i 4 9 ( 9 4 t i 4 + 9 3 × 7 t i 3 + 9 × 2 t i 9 ) = 0 t i 4 + 2 t i 3 + 567 t i 729 = 0 \forall \ i \ 1\leq i \leq 4\begin{cases}{\begin{array}{c}(P\left(\dfrac{-9}{t_i}\right) = 0 \\ \Rightarrow \left(\dfrac{-9}{t_i}\right)^4+7\left(\dfrac{-9}{t_i}\right)^3+2\left(\dfrac{-9}{t_i}\right)-9=0 \\ \Rightarrow \dfrac{9^4}{t_i^4}+\dfrac{-9^3\times 7}{t_i^3}+\dfrac{-9\times 2}{t_i}-9=0 \\ \Rightarrow \dfrac{-t_i^4}{9}\left(\dfrac{9^4}{t_i^4}+\dfrac{-9^3\times 7}{t_i^3}+\dfrac{-9\times 2}{t_i}-9\right)=0 \\ \Rightarrow t_i^4+2t_i^3+567t_i-729=0\end{array}}\end{cases}

Thus sum of coefficients = 1 + 2 + 567 729 = 159 =1+2+567-729=\huge\boxed{-159}

You don't need to do that much work (expanding out the new polynomial). By definition of a polynomial, if we have a polynomial p p , the sum of coefficients of all terms of p p is equal to p ( 1 ) p(1) . We exploit this idea to get our answer.


Denote by e i e_i the i th i^{\textrm{th}} elementary symmetric polynomial made up by the roots of P ( x ) P(x) . By Vieta's formulas, we have,

e 1 = ( 7 ) , e 2 = 0 , e 3 = ( 2 ) , e 4 = ( 9 ) e_1=(-7)~,~e_2=0~,~e_3=(-2)~,~e_4=(-9)

As you noticed, the new four roots in the problem are e 4 / a i e_4/a_i where 1 i 4 1\leq i\leq 4 . As such, our new polynomial (call it g g ) can be written as follows (using Remainder-Factor Theorem):

g ( x ) = k = 1 4 ( x + 9 a k ) g ( 1 ) = k = 1 4 ( 1 + 9 a k ) g ( 1 ) = k = 1 4 ( a k + 9 ) k = 1 4 a k = e 4 + 9 e 3 + 9 2 e 2 + 9 3 e 1 + 9 4 e 4 g(x)=\prod_{k=1}^4\left(x+\frac 9{a_k}\right)\implies g(1)=\prod_{k=1}^4\left(1+\frac 9{a_k}\right)\\ \implies g(1)=\frac{\displaystyle\prod_{k=1}^4(a_k+9)}{\displaystyle\prod_{k=1}^4 a_k}=\frac{e_4+9e_3+9^2e_2+9^3e_1+9^4}{e_4}

Plugging values, we get g ( 1 ) = ( 159 ) g(1)=(-159) which is our answer.

Prasun Biswas - 5 years, 12 months ago

Log in to reply

Good approach. Note that we don't even need to know what e i e_i are. The answer is equal to f ( 9 ) f ( 0 ) \frac{ f( -9) } { f(0) } .

Do you see why?

Calvin Lin Staff - 5 years, 12 months ago

Log in to reply

I guess you meant P ( x ) P(x) instead of f ( x ) f(x) . And the reason is simple. As a consequence of Vieta's Formulas, we can write P ( x ) P(x) as,

P ( x ) = e 0 x 4 e 1 x 3 + e 2 x 2 e 3 x + e 4 ( i ) P(x)=e_0x^4-e_1x^3+e_2x^2-e_3x+e_4\ \ \ \ \ (i)

In the expression for g ( 1 ) g(1) , the numerator matches ( i ) (i) with x = ( 9 ) x=(-9) and the denominator matches ( i ) (i) with x = 0 x=0 . Hence, the answer is P ( 9 ) P ( 0 ) \frac{P(-9)}{P(0)} .

Note: e 0 = 1 ~~~e_0=1 by definition.


I wonder why my previous comment got downvoted 2 times. Makes me sad. :\

EDIT: One of the downvoters suddenly removed his/her downvote. This is becoming weird, to be honest.

Prasun Biswas - 5 years, 12 months ago

Yaar...i jus missed it...sirf answer wrong aaya!!!! Bye the way u r in which class???

sarvesh dubey - 5 years, 12 months ago

Log in to reply

I am in class 10.

Nihar Mahajan - 5 years, 12 months ago

Nihar, in which class you study this way?

Dev Sharma - 5 years, 12 months ago

Log in to reply

Refer this .

Nihar Mahajan - 5 years, 12 months ago

Log in to reply

bro, i am in class 10th too. But i have not studied this way. I guess u r preparing 4 olympiad? Thanks for link.

Dev Sharma - 5 years, 12 months ago

Hey nm, no offence, dont u think this question is overrated??

Aditya Kumar - 5 years, 12 months ago

Log in to reply

Yeah it is overrated. (no offence)

Nihar Mahajan - 5 years, 12 months ago

@Calvin Lin It would be better if you lower down the ratings of this problem. Thanks!

Nihar Mahajan - 5 years, 12 months ago

This is incorrect. The product of the roots is 9 -9 , not 9 9 . The correct polynomial is x 4 + 2 x 3 + 567 x 729 x^4+2x^3+567x-729 .

Patrick Corn - 5 years, 12 months ago

Log in to reply

Sorry , my mistake. I have told Calvin sir to edit the answer.Thanks!

Nihar Mahajan - 5 years, 12 months ago

The answer is right now and I have edited my solution accordingly. Thanks!

Nihar Mahajan - 5 years, 12 months ago
Khor Ec
Jun 17, 2015

For convenience of typing, I would let the roots of original equation be a,b,c and d. Using nature of monic polynomial equation,

∑ a = a+b+c+d = -7

∑ ab = ab+ac+ad+bd+bc+cd = 0

∑ abc = abc+abd+acd+bcd = -2

abcd=-9

To form a new equation with roots abc, abd, acd and bcd, let the roots of new equation be a',b',c' and d'.

∑ a' = ∑ abc = -2

∑ a'b' = ∑ a^2 b^2 c*d = abcd(∑ ab) = -9(0) = 0

∑ a'b'c' = ∑ a^3 b^2 c^2*d^2 = (abcd)^2 (a+b+c+d) = (-9)^2 (-7) = -567

a'b'c'd' = (abcd)^3 = (-9)^3 =-729

Using nature of monic polynomial equation,

the coefficient of x^4 is 1

the coefficient of x^3 is 2

the coefficient of x^2 is 0

the coefficient of x is 567

the constant is -729

So, the sum is 1+2+0+567-729= -159

(This is my first solution, sorry if there is any mistake)

It is a pretty well explained solution, but next time would you please format the expressions properly? Just for the sake of unity and coherence...it really does help! There is a Daums online editor linked whenever you post a solution and is quite simple to use.

Aneesh S. - 5 years, 12 months ago

Log in to reply

No problem, thanks :)

Khor EC - 5 years, 12 months ago

0 pending reports

×

Problem Loading...

Note Loading...

Set Loading...